2 An art studio charges $65 per month for art lessons. There is also an additional supply fee of $25 at the time of registration. Which equation best describes the total cost, c, given the number of months, m?​

Answers

Answer 1

Answer:

The equation should be, 65m+25=c

Answer 2

Answer: it’s going to be c=65m+25

Step-by-step explanation:


Related Questions

Answer the question below, and then fill in the blanks if necessary.
Can the distributive property be used to rewrite
8 X (10 – 3)?
Yes
No
If yes, fill in the blanks below.
8 x (10 - 3) = ( , ) - ( , )

Answers

Answer:

Yes

80-24

Step-by-step explanation:

Using distributive property, multiply 8 by 10 and 8 by negative 3.

9(r) = -1 - 77
Evaluate function

Answers

it would be 9(r) = -78

f(x) =2x-4 and g(x) =-3x+2 find: f(g(x))

Answers

Answer:

f(g(x))=6x-8

Step-by-step explanation:

To keep it simple lets say if , for example

[tex]f(x)=2x-4[/tex]

then f(2) would be what?

[tex]f(2)=2(2)-4\\f(2)=0[/tex]

which means we plugged in x=2 in our f(x) this goes for same as the composite function [tex]f(g(x))[/tex] where we plug in g(x) in f(x) and replace x with g(x). I'll show you

[tex]f(x)=2x-4\\g(x)=-3x+2\\[/tex]

now we insert g(x) in f(x) cause we need f(g(x)) which means we replace x with the function g(x).

[tex]f(g(x))=-2(-3x+2)-4[/tex]

[tex]f(g(x))=6x-4-4\\f(g(x))=6x-8[/tex]

so that is our final answer.

Hope i helped

Solve the problem by entering and solving an equation.
A rectangular picture frame has a perimeter of 52 inches. The height of the frame is 12 inches. What is the width of the frame?

Answers

Given : A rectangular picture frame has a perimeter of 52 inches. The height of the frame is 12 inches.

To Find : The width of the frame .

Solution : Let us take the width of the frame be x . So , we know the formula to find the perimeter of rectangle as ;

[tex]\boxed{\red{\bf Perimeter_{rectangle}=2(l+b)}}[/tex]

Where , l is the length of the rectangle and b is the breadth of the rectangle .

⇒ Perimeter = 2 ( l + b ) .

⇒ 52 in. = 2 ( 12 + x ) in.

⇒ 52 in.= 24in. + 2x .

⇒ 2x =( 52 - 24 ) in

⇒ 2x = 28 in.

⇒ x = 28/2 in.

⇒ x = 14 in.

Hence the width of the rectangle is 14 inches.

Oct 16, 12:30:47 PM
Wyatt has 6 cups of yogurt to make smoothies. Each smoothie uses cup of yogurt.
What is the maximum number of whole smoothies Wyatt can make with the yogurt?

Answers

Wyatt would be able to make 6 whole smoothies.
6 Smoothies

Mark Brainliest

Imagining that she’s taking a summer job working at a local entertainment complex your hourly wage is six dollars you will work 40 hours per week and be paid every other week for a total of 16 weeks use the figure provided estimate your gross pay for the summer and each pay period

Answers

6*40 for how much u make a week then divide 16 by 2 for every other week. Then multiply your sum for 6*40 by 16/2

So you would make $1,920

variation does not depend on that of another.
#6 THE SOLUTION FOR F(X) = 3X + 2 WHEN F(X) IS 2 IS

Answers

Answer:

0

*see below incase you worded it wrong

Step-by-step explanation:

then we substitute 2 for f(x)

2 = 3x + 2

0 = 3x

x = 0

UNLESS you mean f(2) is what then that is:

f(2) = 3(2) + 2 = 8

hope that helps!

MO bisects ∠LMN, m∠NMO =6x-20,and 2x+36. Solve for x and find m∠LMO

Answers

Answer:

fwe

Step-by-step explanation:

Which best proves why the expressions 4 (x + 3) + 2 x and 6 (x + 2) must be equivalent expressions?
1.When x = 3, both expressions have a value of 30.
2.When x = 5, both expressions have a value of 42.
3.When x = 1, both expressions have a value of 18, and when x = 8, both expressions have a value of 60.
4.When x = 2, both expressions have a value of 15, and when x = 6, both expressions 5.have a value of 39.

Answers

Answer:

2

Step-by-step explanation:

I think

Help me three questions please really easy I promise answer

Answers

Answer:

1.y=-2x+7

2. no

3.yes

Step-by-step explanation:

Which order pairs are solutions to the equation 4x+2y=4

Answers

Answer:

Slope=2.000/4.000

​ −2.000x−intercept= 22

​  =1y−intercept= 12

=2.00000

Step-by-step explanation:

Rearrange the equation by subtracting what is to the right of the equal sign from both sides of the equation :

                    4*x+2*y-(4)=0

 Pull out like factors :

  4x + 2y - 4  =   2 • (2x + y - 2)

Solve :    2   =  0

This equation has no solution.

A non-zero constant never equals zero.

 Solve   2x+y-2  = 0

Tiger recognizes that we have here an equation of a straight line. Such an equation is usually written y=mx+b ("y=mx+c" in the UK).

"y=mx+b" is the formula of a straight line drawn on Cartesian coordinate system in which "y" is the vertical axis and "x" the horizontal axis.

In this formula :

y tells us how far up the line goes

x tells us how far along

m is the Slope or Gradient i.e. how steep the line is

b is the Y-intercept i.e. where the line crosses the Y axis

The X and Y intercepts and the Slope are called the line properties. We shall now graph the line  2x+y-2  = 0 and calculate its properties

Notice that when x = 0 the value of y is 2/1 so this line "cuts" the y axis at y= 2.00000

 y-intercept = 2/1  =  2.00000

When y = 0 the value of x is 1/1 Our line therefore "cuts" the x axis at x= 1.00000

 x-intercept = 2/2  =  1

Slope is defined as the change in y divided by the change in x. We note that for x=0, the value of y is 2.000 and for x=2.000, the value of y is -2.000. So, for a change of 2.000 in x (The change in x is sometimes referred to as "RUN") we get a change of -2.000 - 2.000 = -4.000 in y. (The change in y is sometimes referred to as "RISE" and the Slope is m = RISE / RUN)

   Slope     = -4.000/2.000 = -2.000

HELP ME ASAP!!! I WILL MARK BRAINLIEST!!

Answers

Its the last one Brand A is more expensive and is 2$ and Brand B is $1.5

Answer:

Brand A ($2) is more expensive than Brand B ($1.50)  (see below)

Step-by-step explanation:

First, you will have to find the slopes of both lines. Slope is y/x, which is the cost/meter or cost per meter.

Using the slope formula:

          y2 — y1

m  =   ---------------

           x2 — x1

Plug in any two points from both lines:

          y2 — y1

m  =   ---------------  = graph 1 (0, 0), (1,2)

           x2 — x1

            2 — 0

m  =   ---------------

             1 — 0

                2

m  =   ---------------  = $2/m

                1

_______graph 2_______

          y2 — y1

m  =   ---------------  = graph 2 (0, 0), (4, 6)

           x2 — x1

            6 — 0

m  =   ---------------

            4 — 0

                6                  3

m  =   ---------------  = -----------  = $3/2 m

                4                  2

Because you are unable to have an exact cost per meter of Brand B, you can use a ratio:

$3  =  $x

2m =  1m

Cross multiply:

3 (1) = 2x

3 = 2x

x = (3 ÷ 2)

x = 1.5

So Brand B is $1.50 per meter while Brand A is $2.00 per meter. Brand B is cheaper.

The following is the prime factorization of which composite number? 2 to the 2nd power times 3 to the 2nd power times 5

120
60
20
180

Answers

2 to the 2nd power times 3 to the second power times 5

4 X 9 X 5 = 36 X 5 = 180

so the answer is 180


hope this helps

g Find the standard deviation for the number of female college students out of 15 that plan to graduate with a STEM degree.

Answers

Complete question is;

In a recent poll of female college students, 70% stated that they plan to graduate with a STEM degree. A math teacher at ODU decides to randomly sample 15 female college students. Find the standard deviation for the number of female college students out of 15 that plan to graduate with a STEM degree.

Answer:

σ = 1.77

Step-by-step explanation:

We are told that 70% stated that they plan to graduate with a STEM degree.

This means that probability of success is; p = 70% = 0.7

A sample of 15 female college students were randomly selected, this means that number of trials is;

n = 15

Now,in binomial distribution formula for standard deviation is;

σ = √(np(1 - p))

Plugging in the relevant values, we have;

σ = √(15 × 0.7 × (1 - 0.7))

σ = √(15 × 0.7 × 0.3)

σ = √3.15

σ ≈ 1.77

Find the value of x. 4^3 = x

Answers

Answer:

x=64

Step-by-step explanation:

The manager of an automobile dealership is considering a new bonus plan designed to increase sales volume. Currently, the mean sales volume is 14 automobiles per month. The manager wants to conduct a research study to see whether the new bonus plan increases sales volume. To collect data on the plan, a sample of sales personnel will be allowed to sell under the new bonus plan for a one-month period.
A. Which form of the null and alternative hypotheses most appropriate for this situation.
B. Comment on the conclusion when H0 cannot be rejected.
1. No evidence that the new plan increases sales.
2. The research hypothesis ? > 24 is supported; the new plan increases sales.
C. Comment on the conclusion when H0 can be rejected.
1. No evidence that the new plan increases sales.
2. The research hypothesis ? > 24 is supported; the new plan increases sales.

Answers

Answer:

A) Null Hypothesis: H0: μ ≤ 14

Alternative hypothesis: Ha: μ > 14

B) If the null hypothesis H0 is not rejected, it means that there is no sufficient evidence to support the claim that new bonus plan increases

sales volume.

C) If the null hypothesis is rejected, it means there is sufficient evidence to support the claim that the new bonus plan increases sales volume

Step-by-step explanation:

A) We are told that the mean sales volume is 14 automobiles per month.

The hypotheses is defined as follows;

Null Hypothesis: H0: μ ≤ 14

Alternative hypothesis: Ha: μ > 14

B) If the null hypothesis H0 is not rejected, it means that there is no sufficient evidence to support the claim that new bonus plan increases

sales volume.

C) If the null hypothesis is rejected, it means there is sufficient evidence to support the claim that the new bonus plan increases sales volume.

How many quarts does a 100 liter tank hold?

Answers

Answer:     84.535 Quartas.

Step-by-step explanation:

Yousuf needs 7 4/9 yards of fabric to make blanket for his mom for Mother’s Day. He found 3 1/3 yards of pink fabric. How much more does he need to finish the blanket
A. 4 1/9
B. 10 7/9
C. 4 3/9
D. 10 1/9

Answers

The answer is B!
Hoped I Helped!
If I’m wrong don’t blame me
He needs A. 4 1/9 of fabric

Given the function h(x) = x2 – 7x + 8, determine the average rate of change of
the function over the interval 2 < x < 10.

Answers

The average rate of change of the function over the interval 2 < x < 10 is 5

How to determine the average rate?

The function is given as:

h(x) = x^2 - 7x + 8

Calculate h(2) and h(10)

h(2) = 2^2 - 7 * 2 + 8

h(2) = -2

h(10) = 10^2 - 7 * 10 + 8

h(10) = 38

The average rate is then calculated as:

Rate = (h(10) - h(2))/(10 - 2)

This gives

Rate = (38 + 2)/(10 - 2)

Evaluate

Rate = 5

Hence, the average rate of change of the function over the interval 2 < x < 10 is 5

Read more about average rate of change at:

https://brainly.com/question/8728504

#SPJ1

Which functions are symmetric with respect to the origin?

y = arcsinx and y = arccosx
y = arccosx and y = arctanx
y = arctanx and y = arccotx
y = arcsinx and y = arctanx

Answers

Answer:

y=arcsinx and y=arctanx

Step-by-step explanation:

The functions that are symmetric with respect to the origin will be y = arcsin x and y = arctan x. Then the correct option is D.

What is a function?

A function is an assertion, concept, or principle that establishes an association between two variables. Functions may be found throughout mathematics and are essential for the development of significant links.

Simply put, inverse trigonometric operations are the opposites of the fundamental trigonometric parameters sine, cosine, secant, cosecant, tangent, and cotangent.

Axial symmetrical is similarity around an axis; an item is internally symmetric if it retains its appearance when turned around an axis.

The functions that are symmetric with respect to the origin will be y = arcsin x and y = arctan x. Then the correct option is D.

The diagram is given below.

More about the function link is given below.

https://brainly.com/question/5245372

#SPJ2

Due today HELP PLEASEEEEE!!!

Answers

Answer:

Oiiiiio ka cha thik cha


12-14 plzzz if u see the picture

Answers

Answer:

I can't see it very well sorry

If an angle measuring 18 ° is bisected to form two new angles , what is the measure of each new angle ?

Answers

Answer:

9 degrees

Step-by-step explanation:

18/2=9

HELP PLEASE PLEASE 50 PTS
The polynomial is missing the coefficient of the third and last terms.
729x3 – 1,458x2y + ____xy2 – ____y3
If the polynomial is factorable using the binomial theorem, what is the coefficient of xy2?
324
648
972
1,944

Answers

Answer:

C

Step-by-step explanation:

Ed2021

The missing coefficient of xy² is 972, option C is correct.

What is Polynomial?

Polynomial is an expression consisting of indeterminates and coefficients, that involves only the operations of addition, subtraction, multiplication, and positive-integer powers of variables

The given polynomial is 729x³ – 1,458x²y + ____xy² – ____y³

To find the missing coefficients, we can use the fact that the polynomial is factorable using the binomial theorem.

According to the binomial theorem, the expansion of (a + b)^3 is:

(a + b)³ = a³ + 3a²b + 3ab² + b³

Comparing this with the given polynomial, we can see that a = 9x and b = -2y.

Substituting these values in the binomial theorem, we get:

(9x - 2y)³ = (9x)³ + 3(9x)²(-2y) + 3(9x)(-2y)² + (-2y)³

Simplifying this expression, we get:

729x³ - 1,458x²y + 972xy² - 8y³

Therefore, the missing coefficient of xy² is 972, option C is correct.

To learn more on Polynomials click:

https://brainly.com/question/11536910

#SPJ3

Caleb earns $2 for each package delivery he makes in addition to $65 per day. He wants to earn more than $87 each day. Use the inequality 2+65≥87 to determine how many packages, p, Caleb must deliver each day. are made of living cells?

Answers

Answer:

Caleb must deliver at least 11 packages a day

P=11

I NEED HELP HERE ASAP, I’m giving brainliest to the right one. I forgot how to solve this:/

Answers

Answer:

28 weeks

Step-by-step explanation:

It says that $42 was 1/3 of her total. So you multiply $42 by 3 and you get $126. You now divide $126 by her weekly allowance, $4.5, and the answer is 28.

1. $42 x 3 = $126

2. $126/$4.5 = 28

3. The answer is Sharon has been saving her allowance for 28 weeks.

Hope this helps!

-Jerc

Answer:

She has been saving her money up for 28 weeks.

Step-by-step explanation:

1. If 42 is 1/3 of her money, then you would multiply 42 by 3 to get the full 3/3 so: 423=126

2. Next, you would divide $126 by $4.50 because that is the total amount of money divided by how much she saved up each week, so: 126/4.5=28 and that gives us the answer which is 28.

34(8x – 6) – 2 = 12 – x

Answers

Answer:

sorry for my handwriting

i think this is the correct answer

А At a concessan stand l hot dog and one hamburger cost $4.One hotdog and
5
Hamburgers cost $13
Find the cost of one hot dog and the cost of one hamburger ?

Answers

Answer:

2.25

Step-by-step explanation:

let x be cost of a hotdog

let y be cost of hamburger

 

so,

 

5x + 4y = 17.75                    eq. 1

4x + 5y = 18.25                    eq. 2

 

manipulate eq. 1 so solve for x:

 

5x = 17.75 - 4y

 

x = 17.75/5 - (4/5)y

 

x = 3.55 - (4/5)y                  eq. 3

 

substitute eq. 3 to eq. 2:

 

4(3.55 - (4/5)y) + 5y = 18.25

 

14.2 - (16/5)y + 5y = 18.25

 

14.2 + (9/5)y = 18.25

 

(9/5)y = 18.25 - 14.2

 

4.05= (9/5)y

 

y = 2.25

 

x = 3.55 - (4/5)(2.25)

 

x = 1.75

 

hotdog costs $1.75

hamburger costs $2.25

What is the determinant of A=
3
1
11
-1

Answers

-14 is the Determinant of given matrices A.

What is the determinant?

A scalar value connected to the square matrix is known as the determinant. If X is a matrix, then |X| or Det is used to indicate the matrix's determinant (X).

Given a Mtrix A:

3       1

11     -1

The formula for the determinant of a matrix:

The determinant of a 2x2 matrix A = [ a b c d ] is |A| = ad - bc. It is simply obtained by cross-multiplying the elements starting from the top left and then subtracting the products.

In our case,

Determinants of A = (3 * -1 ) - (1 * 11)

Determinants of A = -3 -11

Determinants of A = -14

Therefore, the Determinant of given matrices A is -14.

Learn more about determinants here:

https://brainly.com/question/13369636

#SPJ1

Complete question:

What is the determinant of

A=  3       1

       11     -1

Tim and Tom are trying to earn money to buy a new game system over a 3-month period. Tim saved $45.69 each month. If they need a total of $214.53 to buy the game system, how much does Tom need to earn each of the 3 months in order to buy the game system?
A.25.82
B.103.07
C.77.46
D.168.84

Answers

Answer:

A. 25.82

Step-by-step explanation:

(3*Tim)+(3*tom)=214.53

(3*45.69)+(3*tom)=214.53

137.07+(3*tom)=214.53

-137.07+(3*tom)=214.53-137.07

3*tom=77.46

tom/3=77.46/3

tom=25.82

Other Questions
DO CHILDREN NEEDPRIVACY IN THEIR LIVES,SUPPORT? Shelly runs 2.5 miles in 18 minutes. what is her mile time in minutes. Find atleast 5 numbers between 1/2 and 1/3. Please Solve Brainliest and 10 points! depth of water in a swimming pool decreases by 1/8 inch per day do to evaporation no water is added to the pool for a two week. What is the total change in depth of the water during this time period? PLEASE HELP! ASAP! 14 POINTS!How did the U.S. government use propaganda posters to encourage Americans to support the war effort during WWI? White Fang saw his mother taken aboard Three Eagles canoe, and tried to follow her. A blow from Three Eagles knocked him backward to the land. The canoe shoved off. He sprang into the water and swam after it, deaf to the sharp cries of Grey Beaver to return.This is an example of conflict betweenGrey Beaver and Three Eagles.White Fang and nature.White Fang and Three Eagles.White Fang and Lip-lip. m3 = 57. Find m1. (I'm just typing here to meet the 20 character min.) The equilibrium constant _____________. For an ___________ reaction, heat is a reactant. An increase in temperature and heat favors the ____________ reaction and the value of K c _____________. Sarah has $12.45 in quarters and nickels. She has 39 more nickels than quarters.How many of each coin does she have? The shoes cost 35.00 and the tax is 6.5%. How many tax will you pay on the shoes? The Titanic Shipbuilding Company has a noncancelable contract to build a small cargo vessel. Construction involves a cash outlay of $273,000 at the end of each of the next two years. At the end of the third year the company will receive payment of $650,000. Assume the IRR of this option exceeds the cost of capital. The company can speed up construction by working an extra shift. In this case there will be a cash outlay of $595,000 at the end of the first year followed by a cash payment of $650,000 at the end of the second year. Use the IRR rule to show the (approximate) range of opportunity costs of capital at which the company should work the extra shift.The company should work the extra shift if the cost of capital is between ___________ % and ___________ % Adriano runs a website that helps writers write better stories. Every month, Adriano receives a subscription fee of $5 from each of the subscribers to his website. The website had 200 subscribers last month. This month 40 new members joined the website and 10 members cancelled their subscription. How much money did Adrianos online business earn this month Help me please!! Drawing from the following Spanish building blocks, and the model sentences you have worked with in this lesson, string together at least ten statements. Understanding how a volcano erupts Select the three results of the French and Indian War.The British Empire grew substantially.The Declaratory Act was passed.The colony of Maine was established.American military officers gained valuable experience.Britain began to tax the colonies.Multiple Choice Estimate: 1/4 x 21 Why did Muhammad Ali stance against the Vietnamese war make him a hero to so many African Americans? Kayla rented a limousine for prom. There was a one-time charge of $100, plus an hourly rate of $45. Her total cost for the night was $347.50. How many hours did Kayla rent the limo? please helppp HELP PLEASE & EXPLAIN . *like i said on my other question if you answer my math questions that i post accurately i will give you brainliest each time *Marty's class collected $34.00 in a school-wide penny collection contest. If Tommy's class added $3.00 to the amount they collected,x,and then multiplied the sum by 2, they would have the same amount of money as Marty's class. a) $20.00b) $14.00c) $3.00d) $17.00